Matrixdarstellung des Strahlteilers zur numerischen Berechnung der Ausgabe basierend auf der Eingabe einer gegebenen Photonenzahl (Fock-Zustand).

Frage: Ist es möglich, die Wirkung eines einfachen 50%-Strahlteilers auf Photonenzahlzustände unter Verwendung von Matrizen auszudrücken, so dass die Ausgabe durch Matrizenberechnungen statt durch manuelles Ersetzen von Gleichungen berechnet werden kann?

Um das Problem zu erklären, betrachten Sie einen 50% Strahlteiler und definieren Sie: A 1 , 2 + = Erstellungsoperatoren für Eingabe und B 1 , 2 + = Erstellungsoperatoren für die Ausgabe. Für einen ausgeglichenen 50%-Strahlteiler können wir schreiben, wie der Strahlteiler die Eingabe auf Ausgabeerstellungsoperatoren abbildet:

A 1 + 1 2 ( B 1 + + B 2 + )
A 2 + 1 2 ( B 1 + B 2 + )

Wir können dann die Ausgabe-Photonenzahlverteilung für gegebene Eingabe-Photonenzahlzustände berechnen, indem wir die Fock-Zustände in Form von Erzeugungsoperatoren schreiben, gefolgt von Einsetzen in die obigen Gleichungen.

Betrachten Sie beispielsweise 2 Photonen, die in MODE1 auf den Strahlteiler einfallen, und 1 Photon, das in MODE2 am Eingang einfällt. Wir schreiben sie zuerst als Photonenzahlzustände und drücken sie dann in Form von Erzeugungsoperatoren aus, die auf das Vakuum einwirken.

| ψ ich N = | 2 1 | 1 2 = 1 2 A 1 + 2 A 2 + | 0 1 | 0 2

Wir können den Ausgangszustand durch Umschreiben finden A 1 + & A 2 + bezüglich B 1 + & B 2 + (unter Verwendung der obigen Gleichungen):

| ψ Ö u T = 1 2 2 ( B 1 + + B 2 + ) 2 ( B 1 + B 2 + ) | 0 1 | 0 2

die dann ausgewertet werden können, indem die Klammern erweitert und die Wirkung von Erzeugungsoperatoren auf das Vakuum ausgewertet werden (dh B 1 + | 0 1 = | 1 1 Und B 1 + 2 | 0 1 = 2 | 2 1 usw.). Das Ergebnis dieses Beispiels ist also:

| ψ Ö u T = 1 4 ( 6 | 3 1 | 0 2 + 2 | 2 1 | 1 2 2 | 1 1 | 2 2 6 | 0 1 | 3 2 )

Das obige Verfahren funktioniert, erfordert jedoch eine algebraische Gleichungssubstitution, die keine mathematische Operation ist. Meine Frage ist, wie können wir das Problem so gestalten, dass die Eingabe als Vektor/Matrix eingegeben werden kann und die Ausgabe (z. B. als Vektor, um die Koeffizienten jedes möglichen Photonenzahl-Ausgabezustands anzuzeigen) durch Matrixoperationen berechnet wird, was gut ist geeignet für die Aufführung auf einem Computer. Ich habe den Strahlteilereffekt häufig als Matrix ausgedrückt gesehen: z

B = 1 2 ( 1 1 1 1 )
[ref: https://physics.stackexchange.com/questions/330888/how-to-write-the-output-state-of-a-beam-splitter]

man könnte also eventuell schreiben:

( B 1 + B 2 + ) = 1 2 ( 1 1 1 1 ) ( A 1 + A 2 + )

aber dies drückt die Beziehung nur in Bezug auf einzelne Erstellungsoperatoren aus, und es ist unklar, wie ein Matrixproblem formuliert werden soll, um beliebige Eingaben zu unterstützen (dh wo jede Eingabe mehrere Photonen enthalten kann).

Mögliche Lösungen : Aus meiner Forschung stelle ich fest:

Alle Vorschläge oder Ideen werden sehr geschätzt, danke.

Sicher, es ist möglich, arbeiten Sie es einfach aus, indem Sie die Aktion auf die Schöpfungs-/Vernichtungs-Oepratoren, wie Sie sie oben besprochen haben, in die Fock-Zustände ersetzen. Beantwortet das deine Frage? Oder suchen Sie nach einem einfachen geschlossenen Ausdruck für eine solche Transformation? (Begründete Vermutung: Es kann nicht so schwer sein, es zu erraten und durch Induktion oder ähnliches zu beweisen.)
Danke für den Vorschlag. Freut mich zu hören, dass es möglich ist, aber können Sie bitte ein Beispiel zeigen, da ich nicht ganz verstehe, wie ich das machen würde. Ich suche nach einem Matrixausdruck / -ansatz, mit dem ich ihn in einer gängigen Programmiersprache codieren und dann die Verteilung der Ausgabephotonenzahl für jede Verteilung der Eingabephotonenzahl erhalten kann. Wenn ich beispielsweise die Eingabe als 2 Photonen in MODE1 und 1 Photon in MODE2 wie in meinem Beispiel einfüge, möchte ich, dass die Ausgabe mir die Koeffizienten jedes möglichen Ausgangsphotonenzahlzustands anzeigt (damit ich sie quadrieren kann, um die Wahrscheinlichkeit des Auftretens zu finden ). Danke
Nun, für den Fall von höchstens einem Photon in jedem einfallenden Strahl haben Sie es oben selbst ausgerechnet! (Entschuldigung, erneut überprüft: Nicht genau das, was Sie getan haben, aber Sie können sofort ändern, was Sie für diesen Fall getan haben.)
Ich habe versucht, es auszuarbeiten, kann aber keinen einzigen Ausdruck für die Ausgabe erhalten. Insbesondere ist der Strahlteiler eine 2 x 2-Matrix und wirkt auf einen (2 x 1)-Vektor, was zu einem (2 x 1)-Vektor führt. Das bedeutet, dass die Operatoren b1 und b2 getrennt sind. Es ist jedoch bekannt, dass einige Terme der Ausgabe BEIDE b1 und b2 zusammen erfordern, zB |1>|2>, also wie kann ein (2 x 1) Ausgabevektor diese Information enthalten? Da es 4 mögliche Ausgangszustände gibt (|3>|0>, |2>|1>, |1>|2> und |0>|3>, muss ich die Dimension meines Problems irgendwie erweitern? Entschuldigung, wenn Mir fehlt etwas Offensichtliches? Danke
Tatsächlich möchten Sie zumindest Ihren Ausgaberaum auf den vollen Raum erweitern, den Sie mit 3 Photonen überspannen können, dh die 4 Zustände, die Sie erwähnen. (Sie können den Eingaberaum auch auf diese Dimension erweitern, müssen es aber nicht.) Jedes Photon kann in beide Richtungen gehen, also können Sie a priori eine beliebige Anzahl von Photonen in jeder Ausgabe erhalten (es sei denn, es gibt destruktive Interferenz wie z in HOM.)
Danke für die Hilfe. Könnte ich aus diesem und ersten Kommentar eine Matrix M bilden, so dass: MX = Y, wobei Y der Ausgang und X der Eingangszustand ist. Für mein Beispiel könnte ich X = (0, 1, 0, 0) verwenden, um den Eingangszustand |2>|1> anzuzeigen, und die Ausgabe Y sollte sein: 1/4(√6, √2, -√2, -√6). M muss eine 4 x 4-Matrix sein, die die Umwandlung von Fock-Zuständen in einen Erstellungsoperator und auch die Strahlteileroperation umfasst (wie die 2 x 2-B-Matrix in meinem ursprünglichen Beitrag, aber vielleicht gekachelt entlang der Diagonale einer 4 x 4-Matrix ). Ist das richtig? Oder beinhaltet die (/ eine bessere) Lösung äußere Produkte, um die Dimensionalität zu erweitern?
Ich denke, das macht Sinn. Ich glaube nicht, dass Sie es aufgrund der Kommutierungsbeziehungen (im Grunde des kombinatorischen Faktors sqrt (n)) mit einem einfachen äußeren Produkt schreiben können. Aber ich bin mir sicher, dass es eine schöne geschlossene Form gibt (mit einem kombinatorischen Objekt, einem Binomialkoeffizienten oder ähnlichem).

Antworten (1)

Ich bin mir nicht ganz sicher, was du meinst, aber ich denke, das ist einfach zu bewerkstelligen.

Was Sie daran denken müssen, ist, dass Ihr Strahlteiler auf Produkte von Erzeugungsoperatoren als Produkttransformation wirkt. Also, wenn Ihr Strahlteiler durch die Matrix dargestellt wird

(1) U = ( U 11 U 12 U 21 U 22 )
Dann:
A ich J A J U J ich , (2) A 1 A 1 U 11 + A 2 U 21 , A 2 A 2 U 12 + A 2 U 22
und damit auf einen Produktzustand
| In = 1 2 A 1 A 1 A 2 | 0
Sie erhalten die Ausgabe
(3) | aus = 1 2 ich J k A ich A J A k U ich 1 U J 1 U k 2 .
Beachten Sie, dass nichts (1) auf beschränkt 2 × 2 Matrizen. Im Allgemeinen die Indizes ich J k in (3) läuft über out-Modi, also in a 3 -Kanal-Interferometer, von dem sie laufen würden 1 Zu 3 zum Beispiel.

In Ihrem speziellen Beispiel ist der Koeffizient des ordnungsgemäß normalisierten Zustandsausgangszustands 1 3 ! | 3 1 | 0 wird gefunden, indem die Summe auf alle Kombinationen von beschränkt wird ( ich J k ) die den Faktor multiplizieren ( A 1 ) 3 . Hier sind diese offensichtlich ( ich J k ) = ( 1 , 1 , 1 ) also ist der Koeffizient 1 2 U 11 U 11 U 12 = 1 4 . Somit ist der Koeffizient von | 3 1 | 0 2 = 6 / 4 .

Der Koeffizient des ordnungsgemäß normalisierten Zustands 1 2 | 2 1 | 1 2 wird eine Summe von Begriffen mit sein ( ich J k ) = ( 1 , 1 , 2 ) , ( 1 , 2 , 1 ) oder ( 2 , 1 , 1 ) :

1 2 ( U 11 U 11 U 22 + U 11 U 21 U 12 + U 21 U 11 U 12 ) = 1 2 1 2 2 ( 1 + ( 1 ) + ( 1 ) ) = 1 4
also der Koeffizient von | 2 1 | 1 2 Ist 2 4 . Ich habe einen Vorzeichenunterschied zu Ihnen, wahrscheinlich wegen der Art und Weise, wie ich meine Indizes in (2) platziere: Ihre Definition der Transformation könnte die Indizes haben ( ich J ) drehte sich um zu meiner Art, Dinge zu schreiben. Wenn Sie die Indizes umdrehen, erhalten Sie 2 4 .

Beachten Sie, dass in meiner Notation die Erstellungsoperatoren auf Vektoren abgebildet werden

A 1 | 0 | 1 = ( 1 0 ) , A 2 | 0 | 2 = ( 0 1 ) ,
so dass
U | 1 = | 1 1 | U | 1 + | 2 2 | U | 1 , = | 1 U 11 + | 2 U 21 , = A 1 | 0 U 11 + A 2 | 0 U 21
so dass (2) tatsächlich richtig aussieht.

Es gibt mehr...

Sie können das Problem weiter vereinfachen, indem Sie die Schwinger-Darstellung (oder Jordan-Karte ) verwenden.

damit der Drehimpulszustand

| J M ( A 1 ) J + M ( A 2 ) J M ( J + M ) ! ( J M ) ! | 0 .
Somit wird Ihr Eingangszustand auf den Drehimpulszustand abgebildet
| ψ ich N | 3 2 , 1 2 .
Ihre Strahlteilertransformation entspricht dann (siehe Yurke, B., McCall, SL und Klauder, JR, 1986. SU(2) and SU(1, 1) interferometers. Physical Review A, 33(6), p.4033.) zu eine Drehung U = R z ( 0 ) R j ( π / 2 ) R z ( 0 ) damit Sie dann in einem einzigen Schuss schreiben können
U | ψ ich N R j ( π / 2 ) | 3 2 , 1 2 , = M | 3 2 , M D M , 1 / 2 3 / 2 ( 0 , π / 2 , 0 )
Wo D M M ' J ( a , β , γ ) ist eine Wigner-D-Matrix .

Es ist dann einfach, die Ausgabe in Form von Drehimpulszuständen zu erhalten:

| 3 2 , 3 2 ( 3 2 2 ) + | 3 2 , 3 2 ( 1 2 2 ) | 3 2 , 1 2 + 1 2 2 | 3 2 , 1 2 + | 3 2 , 3 2 ( 3 2 2 )
und dann zurück in Besetzungsnummernzustände umwandeln.

Danke für die sehr hilfreiche Antwort. Der erste Ansatz scheint gut zu funktionieren (der zweite mit Dmatrix ist auch nett, aber ich muss mehr lesen, um die Abbildungen der Drehimpulszustände zu verstehen). Meine einzige verbleibende Frage ist: Wie können wir diesen Ansatz mit Eingaben in Überlagerungszuständen verwenden? Betrachten wir ein Beispiel, bei dem sich ein Eingangsphoton in einem Überlagerungszustand über beide Eingangsmodi befindet: |INPUT> = 1/sqrt(2) * (|0>|1> +|1>|0>). Letztendlich möchte ich diese Analyse skalieren, um mehr Modi (2 Zeitabschnitte + 2 Eingabepfade) zu berücksichtigen. Ich denke, ich kann einfach mehr Indizes für mehr Modi verwenden, wie Sie vorschlagen. Danke
@ Qconfused9102 Der beste Ort, um über den Schwinger-Vertreter zu lesen. ist Gordon Bayms Lectures on Quantum Mechanics. Ja, das Übergeben linearer Kombinationen willkürlicher Zustände wird zu einer Belastung, wenn Sie die Multiplikationsmethode anwenden und auf jeden Teil der Kombination separat einwirken müssen, und dann den Koeffizienten der Kombinationen verfolgen. Aus dieser Perspektive die D -Funktionstrick hilft, da das Beispiel in Ihrem Kommentar wie das Drehen wäre 1 2 ( | 1 / 2 , 1 / 2 > + | 1 / 2 , 1 / 2 > ) .
Nochmals vielen Dank dafür. Glauben Sie aus Interesse, dass dieser Ansatz auch mit kohärenten Zuständen funktionieren könnte? dh Ersetzen von Erstellungsoperatoren durch Verschiebungsoperatoren ... entweder im ursprünglichen oder im Jordan-Schwinger-Ansatz?
Dieser Schwinger-Trick ist im Grunde für S U ( N ) Wenn Sie also einen kohärenten Drehimpulszustand erzeugen, dann ja. Im Fall des kohärenten Standardzustands ist es möglich, den kohärenten Zustand mit 2 Moden zu erweitern S U ( 2 ) Unterraum im Grunde unter Verwendung der Jordan-Karte, dh nehmen Sie einen Staat | N 1 N 2 in Ihrem kohärenten 2-Modus-Zustand und stellen Sie sich dies als Drehimpulszustand vor, aber ansonsten bin ich nicht 100% von dem, was Sie tun, indem Sie "Erstellungsoperatoren durch Verschiebungsoperatoren ersetzen". Vermutlich könnte man Drehungen durch Verschiebungen ersetzen...
... und dann würdest du bekommen D -Funktionen für die Gruppe der Übersetzungen (es sind Bessel-Funktionen, denke ich).